Finding the expected number of flips needed for a coin having probability $p$ of landing on heads












1












$begingroup$



A coin, having probability $p$ of landing on heads and probability of $q=1-p$ of landing on tails. It is continuously flipped until at least one head and one tail have been flipped.

a) Find the expected number of flips needed.




This is not part of a homework assignment. I am studying for a final and don't understand the professors solutions. Since this is clearly geometric, I would think the solution would be:
$$E(N)=sum_{i=0}^{infty}ip^{n-1}q+sum_{i=0}^{infty}iq^{n-1}p=frac{1}{q}+frac{1}{p}.$$
However, I am completely wrong. The answer is
$$E(N)=pleft(1+frac{1}{q}right)+qleft(1+frac{1}{p}right).$$
For example, consider we flip for heads first. Then we have
$$E(Nmid H)=p+psum_{i=0}^{infty}np^{n-1}q.$$
I am not sure why this makes sense. I am not entirely sure why we have an added $1$ and a factored $p$, $q$. Could someone carefully explain why it makes sense that this is the right answer?










share|cite|improve this question









New contributor




Mistah White is a new contributor to this site. Take care in asking for clarification, commenting, and answering.
Check out our Code of Conduct.







$endgroup$








  • 2




    $begingroup$
    It's all a question of the first toss. If it is $H$ then you just get one more than the expected time to get a $T$, if it is $T$ then you just get one more than the expected time to get $H$. Your method is incorrect because the expected number of tosses needed to get one of the two is $1$.
    $endgroup$
    – lulu
    Apr 16 at 22:30








  • 3




    $begingroup$
    In both the title and first paragraph it appears there is $0$ chance of landing tails, so you will wait forever.
    $endgroup$
    – Ross Millikan
    Apr 16 at 22:32












  • $begingroup$
    Note: your sums are hard to follow. What's $n$? The upper limit of the sums should be $infty$, the exponent of the probability ought to be a simple function of $i$. Done correctly, your method ought to work (though it's easier to do it the other way).
    $endgroup$
    – lulu
    Apr 16 at 22:58
















1












$begingroup$



A coin, having probability $p$ of landing on heads and probability of $q=1-p$ of landing on tails. It is continuously flipped until at least one head and one tail have been flipped.

a) Find the expected number of flips needed.




This is not part of a homework assignment. I am studying for a final and don't understand the professors solutions. Since this is clearly geometric, I would think the solution would be:
$$E(N)=sum_{i=0}^{infty}ip^{n-1}q+sum_{i=0}^{infty}iq^{n-1}p=frac{1}{q}+frac{1}{p}.$$
However, I am completely wrong. The answer is
$$E(N)=pleft(1+frac{1}{q}right)+qleft(1+frac{1}{p}right).$$
For example, consider we flip for heads first. Then we have
$$E(Nmid H)=p+psum_{i=0}^{infty}np^{n-1}q.$$
I am not sure why this makes sense. I am not entirely sure why we have an added $1$ and a factored $p$, $q$. Could someone carefully explain why it makes sense that this is the right answer?










share|cite|improve this question









New contributor




Mistah White is a new contributor to this site. Take care in asking for clarification, commenting, and answering.
Check out our Code of Conduct.







$endgroup$








  • 2




    $begingroup$
    It's all a question of the first toss. If it is $H$ then you just get one more than the expected time to get a $T$, if it is $T$ then you just get one more than the expected time to get $H$. Your method is incorrect because the expected number of tosses needed to get one of the two is $1$.
    $endgroup$
    – lulu
    Apr 16 at 22:30








  • 3




    $begingroup$
    In both the title and first paragraph it appears there is $0$ chance of landing tails, so you will wait forever.
    $endgroup$
    – Ross Millikan
    Apr 16 at 22:32












  • $begingroup$
    Note: your sums are hard to follow. What's $n$? The upper limit of the sums should be $infty$, the exponent of the probability ought to be a simple function of $i$. Done correctly, your method ought to work (though it's easier to do it the other way).
    $endgroup$
    – lulu
    Apr 16 at 22:58














1












1








1


1



$begingroup$



A coin, having probability $p$ of landing on heads and probability of $q=1-p$ of landing on tails. It is continuously flipped until at least one head and one tail have been flipped.

a) Find the expected number of flips needed.




This is not part of a homework assignment. I am studying for a final and don't understand the professors solutions. Since this is clearly geometric, I would think the solution would be:
$$E(N)=sum_{i=0}^{infty}ip^{n-1}q+sum_{i=0}^{infty}iq^{n-1}p=frac{1}{q}+frac{1}{p}.$$
However, I am completely wrong. The answer is
$$E(N)=pleft(1+frac{1}{q}right)+qleft(1+frac{1}{p}right).$$
For example, consider we flip for heads first. Then we have
$$E(Nmid H)=p+psum_{i=0}^{infty}np^{n-1}q.$$
I am not sure why this makes sense. I am not entirely sure why we have an added $1$ and a factored $p$, $q$. Could someone carefully explain why it makes sense that this is the right answer?










share|cite|improve this question









New contributor




Mistah White is a new contributor to this site. Take care in asking for clarification, commenting, and answering.
Check out our Code of Conduct.







$endgroup$





A coin, having probability $p$ of landing on heads and probability of $q=1-p$ of landing on tails. It is continuously flipped until at least one head and one tail have been flipped.

a) Find the expected number of flips needed.




This is not part of a homework assignment. I am studying for a final and don't understand the professors solutions. Since this is clearly geometric, I would think the solution would be:
$$E(N)=sum_{i=0}^{infty}ip^{n-1}q+sum_{i=0}^{infty}iq^{n-1}p=frac{1}{q}+frac{1}{p}.$$
However, I am completely wrong. The answer is
$$E(N)=pleft(1+frac{1}{q}right)+qleft(1+frac{1}{p}right).$$
For example, consider we flip for heads first. Then we have
$$E(Nmid H)=p+psum_{i=0}^{infty}np^{n-1}q.$$
I am not sure why this makes sense. I am not entirely sure why we have an added $1$ and a factored $p$, $q$. Could someone carefully explain why it makes sense that this is the right answer?







probability expected-value






share|cite|improve this question









New contributor




Mistah White is a new contributor to this site. Take care in asking for clarification, commenting, and answering.
Check out our Code of Conduct.











share|cite|improve this question









New contributor




Mistah White is a new contributor to this site. Take care in asking for clarification, commenting, and answering.
Check out our Code of Conduct.









share|cite|improve this question




share|cite|improve this question








edited Apr 17 at 16:36







Mistah White













New contributor




Mistah White is a new contributor to this site. Take care in asking for clarification, commenting, and answering.
Check out our Code of Conduct.









asked Apr 16 at 22:28









Mistah WhiteMistah White

62




62




New contributor




Mistah White is a new contributor to this site. Take care in asking for clarification, commenting, and answering.
Check out our Code of Conduct.





New contributor





Mistah White is a new contributor to this site. Take care in asking for clarification, commenting, and answering.
Check out our Code of Conduct.






Mistah White is a new contributor to this site. Take care in asking for clarification, commenting, and answering.
Check out our Code of Conduct.








  • 2




    $begingroup$
    It's all a question of the first toss. If it is $H$ then you just get one more than the expected time to get a $T$, if it is $T$ then you just get one more than the expected time to get $H$. Your method is incorrect because the expected number of tosses needed to get one of the two is $1$.
    $endgroup$
    – lulu
    Apr 16 at 22:30








  • 3




    $begingroup$
    In both the title and first paragraph it appears there is $0$ chance of landing tails, so you will wait forever.
    $endgroup$
    – Ross Millikan
    Apr 16 at 22:32












  • $begingroup$
    Note: your sums are hard to follow. What's $n$? The upper limit of the sums should be $infty$, the exponent of the probability ought to be a simple function of $i$. Done correctly, your method ought to work (though it's easier to do it the other way).
    $endgroup$
    – lulu
    Apr 16 at 22:58














  • 2




    $begingroup$
    It's all a question of the first toss. If it is $H$ then you just get one more than the expected time to get a $T$, if it is $T$ then you just get one more than the expected time to get $H$. Your method is incorrect because the expected number of tosses needed to get one of the two is $1$.
    $endgroup$
    – lulu
    Apr 16 at 22:30








  • 3




    $begingroup$
    In both the title and first paragraph it appears there is $0$ chance of landing tails, so you will wait forever.
    $endgroup$
    – Ross Millikan
    Apr 16 at 22:32












  • $begingroup$
    Note: your sums are hard to follow. What's $n$? The upper limit of the sums should be $infty$, the exponent of the probability ought to be a simple function of $i$. Done correctly, your method ought to work (though it's easier to do it the other way).
    $endgroup$
    – lulu
    Apr 16 at 22:58








2




2




$begingroup$
It's all a question of the first toss. If it is $H$ then you just get one more than the expected time to get a $T$, if it is $T$ then you just get one more than the expected time to get $H$. Your method is incorrect because the expected number of tosses needed to get one of the two is $1$.
$endgroup$
– lulu
Apr 16 at 22:30






$begingroup$
It's all a question of the first toss. If it is $H$ then you just get one more than the expected time to get a $T$, if it is $T$ then you just get one more than the expected time to get $H$. Your method is incorrect because the expected number of tosses needed to get one of the two is $1$.
$endgroup$
– lulu
Apr 16 at 22:30






3




3




$begingroup$
In both the title and first paragraph it appears there is $0$ chance of landing tails, so you will wait forever.
$endgroup$
– Ross Millikan
Apr 16 at 22:32






$begingroup$
In both the title and first paragraph it appears there is $0$ chance of landing tails, so you will wait forever.
$endgroup$
– Ross Millikan
Apr 16 at 22:32














$begingroup$
Note: your sums are hard to follow. What's $n$? The upper limit of the sums should be $infty$, the exponent of the probability ought to be a simple function of $i$. Done correctly, your method ought to work (though it's easier to do it the other way).
$endgroup$
– lulu
Apr 16 at 22:58




$begingroup$
Note: your sums are hard to follow. What's $n$? The upper limit of the sums should be $infty$, the exponent of the probability ought to be a simple function of $i$. Done correctly, your method ought to work (though it's easier to do it the other way).
$endgroup$
– lulu
Apr 16 at 22:58










2 Answers
2






active

oldest

votes


















5












$begingroup$

If you get a head with probability $p$ then the expected number of throws is $1+E(X)$ where $X$ is a geometric distribution requiring a tail to be thrown with probability $q$ so $1+E(X)=1+frac1q$. Similarly if you throw a tail with probability $q$ then the expected number of throws is $1+E(Y)$ where $Y$ is a geometric distribution requiring a head to be thrown with probability $p$ so $1+E(Y)=1+frac1p$. This means that the overall expected number of throws is
$$pleft(1+frac1qright)+qleft(1+frac1pright)$$
because there is a probability $p$ that the expected number of throws is given by $1+E(X)$ and probability $q$ that it is given by $1+E(Y)$.






share|cite|improve this answer











$endgroup$





















    2












    $begingroup$

    Let $X$ be the time of the first head, and $Y$ the time of the first tail, and $W$ the first time when a head and a tail has been flipped.



    You are right in assuming that $E[X]=frac{1}{p}$ and $E[Y]=frac{1}{q}$, But you are wrong in assuming that $W=X+Y$, that's simply not true, actually $W=max(X,Y)$.



    A possible approach. Let $A$ be the indicator variable of the event: "first coin was a head" (hence $X=1$).



    Then use $$E[W]=E[E[W | A ]] = P(A=1) E[W|A=1]+P(A=0) E[W|A=0]=\=p(E[Y]+1)+q(E[X]+1)$$






    share|cite|improve this answer











    $endgroup$














      Your Answer








      StackExchange.ready(function() {
      var channelOptions = {
      tags: "".split(" "),
      id: "69"
      };
      initTagRenderer("".split(" "), "".split(" "), channelOptions);

      StackExchange.using("externalEditor", function() {
      // Have to fire editor after snippets, if snippets enabled
      if (StackExchange.settings.snippets.snippetsEnabled) {
      StackExchange.using("snippets", function() {
      createEditor();
      });
      }
      else {
      createEditor();
      }
      });

      function createEditor() {
      StackExchange.prepareEditor({
      heartbeatType: 'answer',
      autoActivateHeartbeat: false,
      convertImagesToLinks: true,
      noModals: true,
      showLowRepImageUploadWarning: true,
      reputationToPostImages: 10,
      bindNavPrevention: true,
      postfix: "",
      imageUploader: {
      brandingHtml: "Powered by u003ca class="icon-imgur-white" href="https://imgur.com/"u003eu003c/au003e",
      contentPolicyHtml: "User contributions licensed under u003ca href="https://creativecommons.org/licenses/by-sa/3.0/"u003ecc by-sa 3.0 with attribution requiredu003c/au003e u003ca href="https://stackoverflow.com/legal/content-policy"u003e(content policy)u003c/au003e",
      allowUrls: true
      },
      noCode: true, onDemand: true,
      discardSelector: ".discard-answer"
      ,immediatelyShowMarkdownHelp:true
      });


      }
      });






      Mistah White is a new contributor. Be nice, and check out our Code of Conduct.










      draft saved

      draft discarded


















      StackExchange.ready(
      function () {
      StackExchange.openid.initPostLogin('.new-post-login', 'https%3a%2f%2fmath.stackexchange.com%2fquestions%2f3190435%2ffinding-the-expected-number-of-flips-needed-for-a-coin-having-probability-p-of%23new-answer', 'question_page');
      }
      );

      Post as a guest















      Required, but never shown

























      2 Answers
      2






      active

      oldest

      votes








      2 Answers
      2






      active

      oldest

      votes









      active

      oldest

      votes






      active

      oldest

      votes









      5












      $begingroup$

      If you get a head with probability $p$ then the expected number of throws is $1+E(X)$ where $X$ is a geometric distribution requiring a tail to be thrown with probability $q$ so $1+E(X)=1+frac1q$. Similarly if you throw a tail with probability $q$ then the expected number of throws is $1+E(Y)$ where $Y$ is a geometric distribution requiring a head to be thrown with probability $p$ so $1+E(Y)=1+frac1p$. This means that the overall expected number of throws is
      $$pleft(1+frac1qright)+qleft(1+frac1pright)$$
      because there is a probability $p$ that the expected number of throws is given by $1+E(X)$ and probability $q$ that it is given by $1+E(Y)$.






      share|cite|improve this answer











      $endgroup$


















        5












        $begingroup$

        If you get a head with probability $p$ then the expected number of throws is $1+E(X)$ where $X$ is a geometric distribution requiring a tail to be thrown with probability $q$ so $1+E(X)=1+frac1q$. Similarly if you throw a tail with probability $q$ then the expected number of throws is $1+E(Y)$ where $Y$ is a geometric distribution requiring a head to be thrown with probability $p$ so $1+E(Y)=1+frac1p$. This means that the overall expected number of throws is
        $$pleft(1+frac1qright)+qleft(1+frac1pright)$$
        because there is a probability $p$ that the expected number of throws is given by $1+E(X)$ and probability $q$ that it is given by $1+E(Y)$.






        share|cite|improve this answer











        $endgroup$
















          5












          5








          5





          $begingroup$

          If you get a head with probability $p$ then the expected number of throws is $1+E(X)$ where $X$ is a geometric distribution requiring a tail to be thrown with probability $q$ so $1+E(X)=1+frac1q$. Similarly if you throw a tail with probability $q$ then the expected number of throws is $1+E(Y)$ where $Y$ is a geometric distribution requiring a head to be thrown with probability $p$ so $1+E(Y)=1+frac1p$. This means that the overall expected number of throws is
          $$pleft(1+frac1qright)+qleft(1+frac1pright)$$
          because there is a probability $p$ that the expected number of throws is given by $1+E(X)$ and probability $q$ that it is given by $1+E(Y)$.






          share|cite|improve this answer











          $endgroup$



          If you get a head with probability $p$ then the expected number of throws is $1+E(X)$ where $X$ is a geometric distribution requiring a tail to be thrown with probability $q$ so $1+E(X)=1+frac1q$. Similarly if you throw a tail with probability $q$ then the expected number of throws is $1+E(Y)$ where $Y$ is a geometric distribution requiring a head to be thrown with probability $p$ so $1+E(Y)=1+frac1p$. This means that the overall expected number of throws is
          $$pleft(1+frac1qright)+qleft(1+frac1pright)$$
          because there is a probability $p$ that the expected number of throws is given by $1+E(X)$ and probability $q$ that it is given by $1+E(Y)$.







          share|cite|improve this answer














          share|cite|improve this answer



          share|cite|improve this answer








          edited Apr 16 at 22:53

























          answered Apr 16 at 22:42









          Peter ForemanPeter Foreman

          8,5171321




          8,5171321























              2












              $begingroup$

              Let $X$ be the time of the first head, and $Y$ the time of the first tail, and $W$ the first time when a head and a tail has been flipped.



              You are right in assuming that $E[X]=frac{1}{p}$ and $E[Y]=frac{1}{q}$, But you are wrong in assuming that $W=X+Y$, that's simply not true, actually $W=max(X,Y)$.



              A possible approach. Let $A$ be the indicator variable of the event: "first coin was a head" (hence $X=1$).



              Then use $$E[W]=E[E[W | A ]] = P(A=1) E[W|A=1]+P(A=0) E[W|A=0]=\=p(E[Y]+1)+q(E[X]+1)$$






              share|cite|improve this answer











              $endgroup$


















                2












                $begingroup$

                Let $X$ be the time of the first head, and $Y$ the time of the first tail, and $W$ the first time when a head and a tail has been flipped.



                You are right in assuming that $E[X]=frac{1}{p}$ and $E[Y]=frac{1}{q}$, But you are wrong in assuming that $W=X+Y$, that's simply not true, actually $W=max(X,Y)$.



                A possible approach. Let $A$ be the indicator variable of the event: "first coin was a head" (hence $X=1$).



                Then use $$E[W]=E[E[W | A ]] = P(A=1) E[W|A=1]+P(A=0) E[W|A=0]=\=p(E[Y]+1)+q(E[X]+1)$$






                share|cite|improve this answer











                $endgroup$
















                  2












                  2








                  2





                  $begingroup$

                  Let $X$ be the time of the first head, and $Y$ the time of the first tail, and $W$ the first time when a head and a tail has been flipped.



                  You are right in assuming that $E[X]=frac{1}{p}$ and $E[Y]=frac{1}{q}$, But you are wrong in assuming that $W=X+Y$, that's simply not true, actually $W=max(X,Y)$.



                  A possible approach. Let $A$ be the indicator variable of the event: "first coin was a head" (hence $X=1$).



                  Then use $$E[W]=E[E[W | A ]] = P(A=1) E[W|A=1]+P(A=0) E[W|A=0]=\=p(E[Y]+1)+q(E[X]+1)$$






                  share|cite|improve this answer











                  $endgroup$



                  Let $X$ be the time of the first head, and $Y$ the time of the first tail, and $W$ the first time when a head and a tail has been flipped.



                  You are right in assuming that $E[X]=frac{1}{p}$ and $E[Y]=frac{1}{q}$, But you are wrong in assuming that $W=X+Y$, that's simply not true, actually $W=max(X,Y)$.



                  A possible approach. Let $A$ be the indicator variable of the event: "first coin was a head" (hence $X=1$).



                  Then use $$E[W]=E[E[W | A ]] = P(A=1) E[W|A=1]+P(A=0) E[W|A=0]=\=p(E[Y]+1)+q(E[X]+1)$$







                  share|cite|improve this answer














                  share|cite|improve this answer



                  share|cite|improve this answer








                  edited Apr 16 at 22:50

























                  answered Apr 16 at 22:43









                  leonbloyleonbloy

                  42.6k647108




                  42.6k647108






















                      Mistah White is a new contributor. Be nice, and check out our Code of Conduct.










                      draft saved

                      draft discarded


















                      Mistah White is a new contributor. Be nice, and check out our Code of Conduct.













                      Mistah White is a new contributor. Be nice, and check out our Code of Conduct.












                      Mistah White is a new contributor. Be nice, and check out our Code of Conduct.
















                      Thanks for contributing an answer to Mathematics Stack Exchange!


                      • Please be sure to answer the question. Provide details and share your research!

                      But avoid



                      • Asking for help, clarification, or responding to other answers.

                      • Making statements based on opinion; back them up with references or personal experience.


                      Use MathJax to format equations. MathJax reference.


                      To learn more, see our tips on writing great answers.




                      draft saved


                      draft discarded














                      StackExchange.ready(
                      function () {
                      StackExchange.openid.initPostLogin('.new-post-login', 'https%3a%2f%2fmath.stackexchange.com%2fquestions%2f3190435%2ffinding-the-expected-number-of-flips-needed-for-a-coin-having-probability-p-of%23new-answer', 'question_page');
                      }
                      );

                      Post as a guest















                      Required, but never shown





















































                      Required, but never shown














                      Required, but never shown












                      Required, but never shown







                      Required, but never shown

































                      Required, but never shown














                      Required, but never shown












                      Required, but never shown







                      Required, but never shown







                      Popular posts from this blog

                      Plaza Victoria

                      In PowerPoint, is there a keyboard shortcut for bulleted / numbered list?

                      How to put 3 figures in Latex with 2 figures side by side and 1 below these side by side images but in...